- PowerScore Staff
- Posts: 5972
- Joined: Mar 25, 2011
- Fri Feb 02, 2018 4:52 pm
#43438
Complete Question Explanation
(The complete setup for this game can be found here: lsat/viewtopic.php?t=16399)
The correct answer choice is (B)
Answer choice (A) is incorrect because N and T cannot be selected together.
Answer choice (B) is incorrect because F and H cannot be selected together.
Answer choice (C) is incorrect because if K is selected then N must be selected.
Answer choice (D) is the correct answer choice.
Answer choice (E) is incorrect because if H is selected then K must be selected.
(The complete setup for this game can be found here: lsat/viewtopic.php?t=16399)
The correct answer choice is (B)
Answer choice (A) is incorrect because N and T cannot be selected together.
Answer choice (B) is incorrect because F and H cannot be selected together.
Answer choice (C) is incorrect because if K is selected then N must be selected.
Answer choice (D) is the correct answer choice.
Answer choice (E) is incorrect because if H is selected then K must be selected.
Dave Killoran
PowerScore Test Preparation
Follow me on X/Twitter at http://twitter.com/DaveKilloran
My LSAT Articles: http://blog.powerscore.com/lsat/author/dave-killoran
PowerScore Podcast: http://www.powerscore.com/lsat/podcast/
PowerScore Test Preparation
Follow me on X/Twitter at http://twitter.com/DaveKilloran
My LSAT Articles: http://blog.powerscore.com/lsat/author/dave-killoran
PowerScore Podcast: http://www.powerscore.com/lsat/podcast/